Mathcenter Forum  

Go Back   Mathcenter Forum > คณิตศาสตร์โอลิมปิก และอุดมศึกษา > ข้อสอบโอลิมปิก
สมัครสมาชิก คู่มือการใช้ รายชื่อสมาชิก ปฏิทิน ค้นหา ข้อความวันนี้ ทำเครื่องหมายอ่านทุกห้องแล้ว

ตั้งหัวข้อใหม่ Reply
 
เครื่องมือของหัวข้อ ค้นหาในหัวข้อนี้
  #1  
Old 04 สิงหาคม 2013, 16:05
gon's Avatar
gon gon ไม่อยู่ในระบบ
ผู้พิทักษ์กฎขั้นสูง
 
วันที่สมัครสมาชิก: 29 มีนาคม 2001
ข้อความ: 4,605
gon is on a distinguished road
Smile ข้อสอบ SMO 2013

มีเฉพาะของวันแรกนะครับ จะทยอยพิมพ์จนครบครับ มี 4 ข้อ


ข้อสอบ SMO 2013

10th Southeast Mathematical Olympiad

First Day 2013/07/27 08:00-12.00

Yingtan No1. Highschool. Yingtan, Jiangxi China

------------------------------------------------------------------------------------------------------------------------

1. (ENG)Let a, b be real numbers. The equation $x^3-ax^2+bx-a = 0$ has three real roots.

Find the minimum value of $\frac{2a^3-3ab+3a}{b+1}$

1. (ไทย) กำหนดให้ a, b เป็นจำนวนจริง สมการ $x^3-ax^2+bx-a = 0$ มีรากที่เป็นจำนวนจริง 3 ค่า

จงหาต่ำสุดของ $\frac{2a^3-3ab+3a}{b+1}$

3. (ENG) A sequence $\{a_n\}$ sastisfies that

$a_1=1, a_2 = 2, a_{n+1} = \frac{a_n^2+(-1)^n}{a_{n-1}}$ , $(n=2, 3, ...)$

Prove that the sum of the square of any two numbers in the adjacent terms is in this sequence.

3. (ไทย) ลำดับ $\{a_n\}$ ซึ่งสอดคล้องกับ

$a_1=1, a_2 = 2, a_{n+1} = \frac{a_n^2+(-1)^n}{a_{n-1}}$ , $(n=2, 3, ...)$

จงพิสูจน์ว่า ผลบวกของกำลังสองของสองจำนวนใด ๆ ที่อยู่ติดกันในลำดับนี้ จะได้ผลลัพธ์เป็นจำนวนที่อยู่ในลำดับนี้

4. นักกีฬา 12 คน มีหมายเลข 1, 2, 3, ... , 12 ตามลำดับ ถูกแบ่งออกเป็น 2 กลุ่ม คือ A กับ B โดยแต่ละกลุ่มมี 6 คน นักกีฬาในกลุ่ม A ยืนเป็นรูปวงกลมวงหนึ่ง และ นักกีฬาแต่ละคนในกลุ่ม B จะยืนบนไหล่นักกีฬา 2 คนที่ยืนติดกันของกลุ่ม A ถ้าหมายเลขของนักกีฬาแต่ละคนในกลุ่ม B เท่ากับผลรวมของหมายเลขของนักกีฬาสองคนที่ยืนอยู่ใต้เท้าเขาเสมอ จะเรียกการยืนแบบนี้ว่า หอคอย จงหาจำนวนหอคอยทั้งหมดที่แตกต่างกัน

(วงกลมที่จัดในทิศตามเข็มนาฬิกาหรือทวนเข็มนาฬิกา นับเป็นหอคอยแบบเดียวกัน ตัวอย่างเช่นนักกีฬา 8 คน เราเขียนแสดงหมายเลขของนักกีฬากลุ่ม A อยู่ด้านในวงกลม และหมายเลขของนักกีฬากลุ่ม B อยู่ด้านนอกวงกลม ทั้งสามรูปถือว่าเป็นหอคอยเดียวกัน)

Name:  smo_04_firstday.png
Views: 2080
Size:  2.7 KB

22 สิงหาคม 2013 12:44 : ข้อความนี้ถูกแก้ไขแล้ว 2 ครั้ง, ครั้งล่าสุดโดยคุณ gon
เหตุผล: เพิ่มโจทย์
ตอบพร้อมอ้างอิงข้อความนี้
  #2  
Old 08 สิงหาคม 2013, 10:32
Hero13's Avatar
Hero13 Hero13 ไม่อยู่ในระบบ
หัดเดินลมปราณ
 
วันที่สมัครสมาชิก: 19 ธันวาคม 2012
ข้อความ: 34
Hero13 is on a distinguished road
Default

ข้อ1ได้ 12รูท6หรือเปล่าอ่ะครับ
__________________
ปีหน้าเอาใหม่ fight สมาคมคณิต!
ตอบพร้อมอ้างอิงข้อความนี้
  #3  
Old 10 สิงหาคม 2013, 21:01
Suwiwat B's Avatar
Suwiwat B Suwiwat B ไม่อยู่ในระบบ
กระบี่ประสานใจ
 
วันที่สมัครสมาชิก: 30 พฤษภาคม 2008
ข้อความ: 569
Suwiwat B is on a distinguished road
Default

ตอนนี้ได้เเค่ว่าข้อ 2 เเอบๆคิดได้ว่า
$a_n^2 + a_{n+1}^2 = a_{2n+1}$

ปัญหาคือจะเเสดงยังไงดี ??
__________________
ต้องสู้ถึงจะชนะ

CCC Mathematic Fighting

เครียด เลย
ตอบพร้อมอ้างอิงข้อความนี้
  #4  
Old 17 กันยายน 2013, 22:04
kongp kongp ไม่อยู่ในระบบ
ลมปราณไร้สภาพ
 
วันที่สมัครสมาชิก: 05 พฤษภาคม 2006
ข้อความ: 1,127
kongp is on a distinguished road
Default

รู้สึกว่าข้อ 4 ฝรั่งบางท่านเรียกว่าปัญหาดินเนอร์ คงจะเจอในหนังสือเรียนในชื่ออื่นๆ อีก นัยว่าโจทย์ข้อนี้เริ่มจากการหาจุดอ้างอิงก่อนอื่น เพราะถ้านับแบบไม่มีจุดอ้างอิงจะได้จำนวนแบบจำนวนมากเลยละครับ
ตอบพร้อมอ้างอิงข้อความนี้
  #5  
Old 23 กันยายน 2013, 01:06
Thgx0312555's Avatar
Thgx0312555 Thgx0312555 ไม่อยู่ในระบบ
กระบี่ประสานใจ
 
วันที่สมัครสมาชิก: 12 สิงหาคม 2011
ข้อความ: 885
Thgx0312555 is on a distinguished road
Default

ข้อ 1 ต้อง a,b>0 ด้วยหรือเปล่าครับ
เพราะถ้า มี จะได้คำตอบสวยมาก ($9\sqrt{3}$)
ถ้าไม่มีเงื่อนไข คิดว่าหาคำตอบไม่ได้ ($-\infty$)
__________________
----/---~Alice~ จงรับรู้ไว้ ชื่อแห่งสีสันหนึ่งเดียวที่แสดงผล
---/---- ~Blue~ นี่คือ สีแห่งความหลังอันกว้างใหญ่ของเว็บบอร์ดนี้
ตอบพร้อมอ้างอิงข้อความนี้
  #6  
Old 23 กันยายน 2013, 22:01
gon's Avatar
gon gon ไม่อยู่ในระบบ
ผู้พิทักษ์กฎขั้นสูง
 
วันที่สมัครสมาชิก: 29 มีนาคม 2001
ข้อความ: 4,605
gon is on a distinguished road
Default

อ้างอิง:
ข้อความเดิมเขียนโดยคุณ Thgx0312555 View Post
ข้อ 1 ต้อง a,b>0 ด้วยหรือเปล่าครับ
เพราะถ้า มี จะได้คำตอบสวยมาก ($9\sqrt{3}$)
ถ้าไม่มีเงื่อนไข คิดว่าหาคำตอบไม่ได้ ($-\infty$)
ต้นฉบับถ่ายจากข้อสอบจริงไม่มี a, b > 0 เป็นแบบที่พิมพ์ลงไปครับ.

ปล. เจ้าของข้อสอบดูเหมือนว่าจะตอบไป $-9\sqrt{3}$
ตอบพร้อมอ้างอิงข้อความนี้
  #7  
Old 30 กันยายน 2013, 09:19
artty60 artty60 ไม่อยู่ในระบบ
ลมปราณไร้สภาพ
 
วันที่สมัครสมาชิก: 10 พฤศจิกายน 2010
ข้อความ: 1,036
artty60 is on a distinguished road
Default

ข้อ1.จะหายังไงครับท่านgon .ใช้ความสัมพันธ์รากกับสปส.รู้สึกว่ามันยุ่งเหยิงจนตาลาย
ตอบพร้อมอ้างอิงข้อความนี้
  #8  
Old 30 กันยายน 2013, 18:13
gon's Avatar
gon gon ไม่อยู่ในระบบ
ผู้พิทักษ์กฎขั้นสูง
 
วันที่สมัครสมาชิก: 29 มีนาคม 2001
ข้อความ: 4,605
gon is on a distinguished road
Icon16

อ้างอิง:
ข้อความเดิมเขียนโดยคุณ artty60 View Post
ข้อ1.จะหายังไงครับท่านgon .ใช้ความสัมพันธ์รากกับสปส.รู้สึกว่ามันยุ่งเหยิงจนตาลาย
ถ้า $x_1, x_2, x_3$ เป็นรากของสมการกำลังสาม $x^3 + ax^2 + bx + c = 0$

ค่าของ Discriminant คือ $\bigtriangleup = (x_1-x_2)^2(x_1-x_3)^2(x_2-x_3)^2$

ซึ่งจะพิสูจน์ได้ว่า $(x_1-x_2)^2(x_1-x_3)^2(x_2-x_3)^2 = 18abc - 4a^3c + a^2b^2 - 4b^3 - 27c^2$

โดยการพิสูจน์จะเริ่มคิดจากกรณีที่เป็นสมการกำลังสามลดรูป $x^3 + px + q = 0$ จะได้ $\bigtriangleup = -4p^3 - 27q^2 $ จากนั้นเราก็แปลงสมการ $x^3 + ax^2 + bx + c = 0$ โดยสมมติให้ $y = x+\frac{a}{3}$ ก็จะได้สมการกำลังสามลดรูปของตัวแปร y จากนั้นจึงประยุกต์กรณีลดรูปลงไปอีกครั้ง

และเนื่องจากรากของสมการกำลังสามบนจำนวนจริงจะมีได้สามแบบคือ

1. เป็นจำนวนจริงที่ต่างกันทั้งหมด

2. เป็นจำนวนเชิงซ้อนที่เป็นสังยุคกันหนึ่งคู่ กับ จำนวนจริงหนึ่งค่า

3. เป็นจำนวนจริงทั้งหมด แต่มีอยู่อย่างน้อย 2 ค่าที่เท่ากัน

ซึ่งจากรากทั้งสามแบบจะได้ว่า

แบบที่ 1. จะเกิดเมื่อ $\bigtriangleup > 0$

แบบที่ 2. จะเกิดเมื่อ $\bigtriangleup < 0$

แบบที่ 3. จะเกิดเมื่อ $\bigtriangleup = 0$

สำหรับข้อนี้ของเราจึงใช้เงื่อนไขแบบที่ 1+3 นั่นเองครับ.

ปล. ผมยังไม่ได้ยังลองคิดจริงจังครับ ตอนนี้ผมติดพันการอ่านนิยายจีนอยู่

เนื่องจากผมไม่ค่อยมีเวลาเหลือแต่ละวันเท่าไร เสาร์ที่ผ่านมาผมพอมีเวลาเลยแวบไปซื้อนิยายจีนที่คิดว่าจะซื้อมา 11 เล่ม

เพิ่งอ่านจบไป 3 เล่ม ยังขาดอีก 8 เล่ม หรืิอประมาณ 1500- 2000 หน้า

เดี๋ยวจะหาเวลามาคิดต่อภายหลังครับ.

30 กันยายน 2013 18:21 : ข้อความนี้ถูกแก้ไขแล้ว 1 ครั้ง, ครั้งล่าสุดโดยคุณ gon
เหตุผล: +3
ตอบพร้อมอ้างอิงข้อความนี้
  #9  
Old 30 กันยายน 2013, 18:29
artty60 artty60 ไม่อยู่ในระบบ
ลมปราณไร้สภาพ
 
วันที่สมัครสมาชิก: 10 พฤศจิกายน 2010
ข้อความ: 1,036
artty60 is on a distinguished road
Default

ขอบคุณท่านgonมากที่ให้คำชี้แนะ
ตอบพร้อมอ้างอิงข้อความนี้
  #10  
Old 01 ตุลาคม 2013, 20:56
Thgx0312555's Avatar
Thgx0312555 Thgx0312555 ไม่อยู่ในระบบ
กระบี่ประสานใจ
 
วันที่สมัครสมาชิก: 12 สิงหาคม 2011
ข้อความ: 885
Thgx0312555 is on a distinguished road
Default

ข้อ 1 ลดเหลือแค่ $a^2 \ge 3b$ ให้ได้ครับ

แล้วก็จะได้ $2a^3-3ab+3a \ge 3ab+3a$ ...
__________________
----/---~Alice~ จงรับรู้ไว้ ชื่อแห่งสีสันหนึ่งเดียวที่แสดงผล
---/---- ~Blue~ นี่คือ สีแห่งความหลังอันกว้างใหญ่ของเว็บบอร์ดนี้
ตอบพร้อมอ้างอิงข้อความนี้
  #11  
Old 06 ตุลาคม 2013, 16:57
ความรู้ยังอ่อนด้อย's Avatar
ความรู้ยังอ่อนด้อย ความรู้ยังอ่อนด้อย ไม่อยู่ในระบบ
จอมยุทธ์หน้าหยก
 
วันที่สมัครสมาชิก: 18 กันยายน 2010
ข้อความ: 175
ความรู้ยังอ่อนด้อย is on a distinguished road
Default

1. ให้ $x_1,x_2,x_3$ เป็นรากของสมการ ทำให้ได้ว่า $x_1+x_2+x_3=x_1x_2x_3$

โดย AM-GM ทำให้ได้ $x_1x_2x_3 \geq 3\sqrt{3}$ จาก $(x_1+x_2+x_3)^2 \geq 3(x_1x_2+x_2x_3+x_3x_1)$

เอาไปแทนในโจทย์ได้ $\dfrac{2a^3-3ab+3b}{b+1} \geq \dfrac{6ab-3ab+3b}{b+1} = 3a$

แต่ $a \geq 3\sqrt{3}$ ดังนั้นค่าต่ำสุดคือ $9\sqrt{3}$ เกิดสมการเมื่อ $x_1=x_2=x_3$

2. $a_n^2+(-1)^n=a_{n+1}a_{n-1}$

แทน n ด้วย n-1 แล้วนำสองสมการมาบวกกันจึงได้

$a_n^2+a_{n-1}^2=a_{n+1}a_{n-1}+a_na_{n-2}$

$\dfrac{a_n}{a_{n-1}} = \dfrac{a_{n+1}-a_{n-1}}{a_n-a_{n-2}}$

จึงได้...

$\dfrac{a_n}{a_2}=\dfrac{a_{n+1}-a_{n-1}}{a_3-a_1}$

แทนค่า $a_1,a_2,a_3$ ลงในสมการ

$a_{n+1}=2a_n+a_{n-1}$

จึงได้ $a_n = \dfrac{1}{2\sqrt{2}}\left(\,(1+\sqrt{2})^n-(1-\sqrt{2})^n\right) $

$a_n^2+a_{n-1}^2= a_{2n-1}$

อีกข้อยังไม่ออกอ่าครับบ
ตอบพร้อมอ้างอิงข้อความนี้
  #12  
Old 27 พฤศจิกายน 2013, 11:30
นกกะเต็นปักหลัก's Avatar
นกกะเต็นปักหลัก นกกะเต็นปักหลัก ไม่อยู่ในระบบ
ลมปราณคุ้มครองร่าง
 
วันที่สมัครสมาชิก: 28 มกราคม 2013
ข้อความ: 288
นกกะเต็นปักหลัก is on a distinguished road
Default

second day
1.Let $f(x)=\left[\,\frac{x}{1!} \right]+\left[\,\frac{x}{2!} \right]+\left[\,\frac{x}{3!} \right]+\cdot \cdot \cdot +\left[\,\frac{x}{2013!} \right]$.For an integer n.If the equation $f(x)=n$ has some real roots, then $n$ is called a "lucky number".How many luckers number are in the set ${1,3,5,\cdot \cdot \cdot ,2013}$
__________________
โลกนี้ช่าง...

27 พฤศจิกายน 2013 11:32 : ข้อความนี้ถูกแก้ไขแล้ว 3 ครั้ง, ครั้งล่าสุดโดยคุณ นกกะเต็นปักหลัก
ตอบพร้อมอ้างอิงข้อความนี้
  #13  
Old 27 พฤศจิกายน 2013, 11:38
นกกะเต็นปักหลัก's Avatar
นกกะเต็นปักหลัก นกกะเต็นปักหลัก ไม่อยู่ในระบบ
ลมปราณคุ้มครองร่าง
 
วันที่สมัครสมาชิก: 28 มกราคม 2013
ข้อความ: 288
นกกะเต็นปักหลัก is on a distinguished road
Default

2.Let $n$ be an integer , and $n>1$. $p_1,p_2,p_3,\cdot \cdot \cdot ,p_n$ denote the first $n$ primes in ascending order ( $p_1=2$, $p_2=3$,$\cdot \cdot \cdot $). Define $A={p_1}^{p_1}{p_2}^{p_2}{p_3}^{p_3}\cdot \cdot \cdot {p_n}^{p_n}$.FInd all positive integers for $x$ ,so that $\frac{A}{x} $is an even number and there are exactly $x$ different positive divisors in $\frac{A}{x} $.
__________________
โลกนี้ช่าง...
ตอบพร้อมอ้างอิงข้อความนี้
  #14  
Old 27 พฤศจิกายน 2013, 11:53
นกกะเต็นปักหลัก's Avatar
นกกะเต็นปักหลัก นกกะเต็นปักหลัก ไม่อยู่ในระบบ
ลมปราณคุ้มครองร่าง
 
วันที่สมัครสมาชิก: 28 มกราคม 2013
ข้อความ: 288
นกกะเต็นปักหลัก is on a distinguished road
Default

3. If any$ 2\times2$ squares are removed from a $3\times 3$ squares, then we get a graph which is called an "angle shape" .(One kind of angle shape is shown in the first figure.)Some non-overlapping angle shapes are placed in a $10\times 10$ square table (another figure), the biundaries of the angle shapes coincide with the boundaries or dividing lines of thwe table,Find the maximum integer $k$ , so that whatever way you place $k$ angle shapes in the table, you can always put one more angle shape inside the table.
รูปภาพที่แนบมาด้วย
   
__________________
โลกนี้ช่าง...
ตอบพร้อมอ้างอิงข้อความนี้
  #15  
Old 27 พฤศจิกายน 2013, 17:48
นกกะเต็นปักหลัก's Avatar
นกกะเต็นปักหลัก นกกะเต็นปักหลัก ไม่อยู่ในระบบ
ลมปราณคุ้มครองร่าง
 
วันที่สมัครสมาชิก: 28 มกราคม 2013
ข้อความ: 288
นกกะเต็นปักหลัก is on a distinguished road
Default

4. Let $n\geqslant 3$, $\alpha ,\beta ,\gamma \in (0,1), a_k,b_k,c_k\geqslant 0 (k=1,2,3,\cdot \cdot \cdot ,n)$ satisfy the following inequalities
$$\sum_{k = 1}^{n} {(k+\alpha )a_k\leqslant a},\sum_{k = 1}^{n} {(k+\beta )b_k\leqslant b},\sum_{k = 1}^{n} {(k+\gamma )c_k\leqslant c}.$$
If for any above $a_k,b_k,c_k(k=1,2,3,\cdot \cdot \cdot ,n),$ we have $\sum_{k = 1}^{n}{(k+\lambda )a_kb_kc_k}\leqslant \lambda $, find the minimum value of $\lambda $
__________________
โลกนี้ช่าง...

27 พฤศจิกายน 2013 17:49 : ข้อความนี้ถูกแก้ไขแล้ว 3 ครั้ง, ครั้งล่าสุดโดยคุณ นกกะเต็นปักหลัก
ตอบพร้อมอ้างอิงข้อความนี้
ตั้งหัวข้อใหม่ Reply


หัวข้อคล้ายคลึงกัน
หัวข้อ ผู้ตั้งหัวข้อ ห้อง คำตอบ ข้อความล่าสุด
ผลการแข่งขันIMO 2013 ฟินิกซ์เหินฟ้า ข่าวคราวแวดวง ม.ปลาย 7 28 กรกฎาคม 2013 11:31
การแข่งขันคณิตศาสตร์ระหว่างโรงเรียน Yorwor Mathematics Competition 2013 (YMC 2013) geophysics ข่าวคราวแวดวงประถม ปลาย 3 26 กรกฎาคม 2013 15:41
ผลการแข่งขัน Emic 2013 (Bimc 2013) @บัลแกเรีย gon ข่าวคราวแวดวงประถม ปลาย 16 23 กรกฎาคม 2013 00:00
ผลการแข่งขัน IWYMIC 2013 (Bimc 2013) @บัลแกเรีย gon ข่าวคราวแวดวง ม.ต้น 13 18 กรกฎาคม 2013 14:41
การแข่งขันคณิตศาสตร์ระหว่างโรงเรียน Yorwor Mathematics Competition 2013 (YMC 2013) geophysics ข่าวคราวแวดวง ม.ต้น 5 08 กรกฎาคม 2013 22:06

เครื่องมือของหัวข้อ ค้นหาในหัวข้อนี้
ค้นหาในหัวข้อนี้:

ค้นหาขั้นสูง

กฎการส่งข้อความ
คุณ ไม่สามารถ ตั้งหัวข้อใหม่ได้
คุณ ไม่สามารถ ตอบหัวข้อได้
คุณ ไม่สามารถ แนบไฟล์และเอกสารได้
คุณ ไม่สามารถ แก้ไขข้อความของคุณเองได้

vB code is On
Smilies are On
[IMG] code is On
HTML code is Off
ทางลัดสู่ห้อง


เวลาที่แสดงทั้งหมด เป็นเวลาที่ประเทศไทย (GMT +7) ขณะนี้เป็นเวลา 06:36


Powered by vBulletin® Copyright ©2000 - 2024, Jelsoft Enterprises Ltd.
Modified by Jetsada Karnpracha